Integral of a vector difference












1














Not sure how to deal with this problem:



$$dfrac {doverrightarrow {h}}{dt}=overrightarrow {r_1}times left( -dfrac {m}{left( overrightarrow {r_2}-overrightarrow {r_1}right) ^{2}}right)$$



where $r_2$ and $r_1$ are vectors. The parameter $t$ represents time. $h$ is the specific angular momentum. The vector direction of the right side term, I suppose, would be $(r_2-r_1)$.



This is as far as I get. But I’m not sure if I’m headed in the right direction:



$$dfrac {doverrightarrow {h}}{dt}=overrightarrow {r_1}times left( -dfrac {m}{left( overrightarrow {r_2}-overrightarrow {r_1}right) ^{3}}left( overrightarrow {r_2}-overrightarrow {r_1}right) right)$$



$$int dfrac {doverrightarrow {h}}{dt} dt=int -dfrac {m}{left( overrightarrow {r_2}-overrightarrow {r_1}right) ^{3}}r_2r_1sin theta dt$$



$$h=int dfrac {mr_2r_1sin theta }{left( left( r_{2}cos theta _{2}-r_{1}cos theta _{1}right) +left( r_{2}sin theta _{2}-r_{1}sin theta _{1}right) right) ^{frac {3}{2}}} dt$$



$$h=dfrac {mr_{2}r_{1}sin theta }{left( overrightarrow {r}_{2}-overrightarrow {r}_{1}right) ^{3}}t+C$$










share|cite|improve this question





























    1














    Not sure how to deal with this problem:



    $$dfrac {doverrightarrow {h}}{dt}=overrightarrow {r_1}times left( -dfrac {m}{left( overrightarrow {r_2}-overrightarrow {r_1}right) ^{2}}right)$$



    where $r_2$ and $r_1$ are vectors. The parameter $t$ represents time. $h$ is the specific angular momentum. The vector direction of the right side term, I suppose, would be $(r_2-r_1)$.



    This is as far as I get. But I’m not sure if I’m headed in the right direction:



    $$dfrac {doverrightarrow {h}}{dt}=overrightarrow {r_1}times left( -dfrac {m}{left( overrightarrow {r_2}-overrightarrow {r_1}right) ^{3}}left( overrightarrow {r_2}-overrightarrow {r_1}right) right)$$



    $$int dfrac {doverrightarrow {h}}{dt} dt=int -dfrac {m}{left( overrightarrow {r_2}-overrightarrow {r_1}right) ^{3}}r_2r_1sin theta dt$$



    $$h=int dfrac {mr_2r_1sin theta }{left( left( r_{2}cos theta _{2}-r_{1}cos theta _{1}right) +left( r_{2}sin theta _{2}-r_{1}sin theta _{1}right) right) ^{frac {3}{2}}} dt$$



    $$h=dfrac {mr_{2}r_{1}sin theta }{left( overrightarrow {r}_{2}-overrightarrow {r}_{1}right) ^{3}}t+C$$










    share|cite|improve this question



























      1












      1








      1







      Not sure how to deal with this problem:



      $$dfrac {doverrightarrow {h}}{dt}=overrightarrow {r_1}times left( -dfrac {m}{left( overrightarrow {r_2}-overrightarrow {r_1}right) ^{2}}right)$$



      where $r_2$ and $r_1$ are vectors. The parameter $t$ represents time. $h$ is the specific angular momentum. The vector direction of the right side term, I suppose, would be $(r_2-r_1)$.



      This is as far as I get. But I’m not sure if I’m headed in the right direction:



      $$dfrac {doverrightarrow {h}}{dt}=overrightarrow {r_1}times left( -dfrac {m}{left( overrightarrow {r_2}-overrightarrow {r_1}right) ^{3}}left( overrightarrow {r_2}-overrightarrow {r_1}right) right)$$



      $$int dfrac {doverrightarrow {h}}{dt} dt=int -dfrac {m}{left( overrightarrow {r_2}-overrightarrow {r_1}right) ^{3}}r_2r_1sin theta dt$$



      $$h=int dfrac {mr_2r_1sin theta }{left( left( r_{2}cos theta _{2}-r_{1}cos theta _{1}right) +left( r_{2}sin theta _{2}-r_{1}sin theta _{1}right) right) ^{frac {3}{2}}} dt$$



      $$h=dfrac {mr_{2}r_{1}sin theta }{left( overrightarrow {r}_{2}-overrightarrow {r}_{1}right) ^{3}}t+C$$










      share|cite|improve this question















      Not sure how to deal with this problem:



      $$dfrac {doverrightarrow {h}}{dt}=overrightarrow {r_1}times left( -dfrac {m}{left( overrightarrow {r_2}-overrightarrow {r_1}right) ^{2}}right)$$



      where $r_2$ and $r_1$ are vectors. The parameter $t$ represents time. $h$ is the specific angular momentum. The vector direction of the right side term, I suppose, would be $(r_2-r_1)$.



      This is as far as I get. But I’m not sure if I’m headed in the right direction:



      $$dfrac {doverrightarrow {h}}{dt}=overrightarrow {r_1}times left( -dfrac {m}{left( overrightarrow {r_2}-overrightarrow {r_1}right) ^{3}}left( overrightarrow {r_2}-overrightarrow {r_1}right) right)$$



      $$int dfrac {doverrightarrow {h}}{dt} dt=int -dfrac {m}{left( overrightarrow {r_2}-overrightarrow {r_1}right) ^{3}}r_2r_1sin theta dt$$



      $$h=int dfrac {mr_2r_1sin theta }{left( left( r_{2}cos theta _{2}-r_{1}cos theta _{1}right) +left( r_{2}sin theta _{2}-r_{1}sin theta _{1}right) right) ^{frac {3}{2}}} dt$$



      $$h=dfrac {mr_{2}r_{1}sin theta }{left( overrightarrow {r}_{2}-overrightarrow {r}_{1}right) ^{3}}t+C$$







      calculus






      share|cite|improve this question















      share|cite|improve this question













      share|cite|improve this question




      share|cite|improve this question








      edited Nov 29 '18 at 23:38







      alexmarison

















      asked Nov 29 '18 at 3:17









      alexmarisonalexmarison

      62




      62






















          0






          active

          oldest

          votes











          Your Answer





          StackExchange.ifUsing("editor", function () {
          return StackExchange.using("mathjaxEditing", function () {
          StackExchange.MarkdownEditor.creationCallbacks.add(function (editor, postfix) {
          StackExchange.mathjaxEditing.prepareWmdForMathJax(editor, postfix, [["$", "$"], ["\\(","\\)"]]);
          });
          });
          }, "mathjax-editing");

          StackExchange.ready(function() {
          var channelOptions = {
          tags: "".split(" "),
          id: "69"
          };
          initTagRenderer("".split(" "), "".split(" "), channelOptions);

          StackExchange.using("externalEditor", function() {
          // Have to fire editor after snippets, if snippets enabled
          if (StackExchange.settings.snippets.snippetsEnabled) {
          StackExchange.using("snippets", function() {
          createEditor();
          });
          }
          else {
          createEditor();
          }
          });

          function createEditor() {
          StackExchange.prepareEditor({
          heartbeatType: 'answer',
          autoActivateHeartbeat: false,
          convertImagesToLinks: true,
          noModals: true,
          showLowRepImageUploadWarning: true,
          reputationToPostImages: 10,
          bindNavPrevention: true,
          postfix: "",
          imageUploader: {
          brandingHtml: "Powered by u003ca class="icon-imgur-white" href="https://imgur.com/"u003eu003c/au003e",
          contentPolicyHtml: "User contributions licensed under u003ca href="https://creativecommons.org/licenses/by-sa/3.0/"u003ecc by-sa 3.0 with attribution requiredu003c/au003e u003ca href="https://stackoverflow.com/legal/content-policy"u003e(content policy)u003c/au003e",
          allowUrls: true
          },
          noCode: true, onDemand: true,
          discardSelector: ".discard-answer"
          ,immediatelyShowMarkdownHelp:true
          });


          }
          });














          draft saved

          draft discarded


















          StackExchange.ready(
          function () {
          StackExchange.openid.initPostLogin('.new-post-login', 'https%3a%2f%2fmath.stackexchange.com%2fquestions%2f3018109%2fintegral-of-a-vector-difference%23new-answer', 'question_page');
          }
          );

          Post as a guest















          Required, but never shown

























          0






          active

          oldest

          votes








          0






          active

          oldest

          votes









          active

          oldest

          votes






          active

          oldest

          votes
















          draft saved

          draft discarded




















































          Thanks for contributing an answer to Mathematics Stack Exchange!


          • Please be sure to answer the question. Provide details and share your research!

          But avoid



          • Asking for help, clarification, or responding to other answers.

          • Making statements based on opinion; back them up with references or personal experience.


          Use MathJax to format equations. MathJax reference.


          To learn more, see our tips on writing great answers.





          Some of your past answers have not been well-received, and you're in danger of being blocked from answering.


          Please pay close attention to the following guidance:


          • Please be sure to answer the question. Provide details and share your research!

          But avoid



          • Asking for help, clarification, or responding to other answers.

          • Making statements based on opinion; back them up with references or personal experience.


          To learn more, see our tips on writing great answers.




          draft saved


          draft discarded














          StackExchange.ready(
          function () {
          StackExchange.openid.initPostLogin('.new-post-login', 'https%3a%2f%2fmath.stackexchange.com%2fquestions%2f3018109%2fintegral-of-a-vector-difference%23new-answer', 'question_page');
          }
          );

          Post as a guest















          Required, but never shown





















































          Required, but never shown














          Required, but never shown












          Required, but never shown







          Required, but never shown

































          Required, but never shown














          Required, but never shown












          Required, but never shown







          Required, but never shown







          Popular posts from this blog

          Quarter-circle Tiles

          build a pushdown automaton that recognizes the reverse language of a given pushdown automaton?

          Mont Emei